Discussion

Which one of the following is an acceptable placement of dishes on the bottom, middle, and top shelves?
(A)bottom: dish 1 middle: dish 6 top: dishes 2, 3, 4, 5
(B)...
(C)...
(D)...
(E)...
(F)...
*This question is included in June 2013 LSAT (PT69): Analytical Reasoning, question #6

The solution is

Image Not Available
Posted: 10/20/2013 08:34
I paid money for this LSAT. Which of the following is true? The app developers, or the test preparers are dumber? The questions are about the manuscripts but the answers are about the Petri dishes!
Image Not Available
Contributor
Posted: 10/20/2013 19:34
Hi B A, let us take a look into this. Sorry about the inconvenience, and we'll get back to you soon. Next time when you post, please mention the test number, section, and problem number.
Posted: 10/20/2013 20:10
There was an error in loading the questions and answers to the app. We are in the process of fixing it and it should be fixed in the next update (about 1 week). If you can't wait that long, we will send you a refund by PayPal. Please email support@arcadiaprep.com

You need to be signed in to perform that action.

Sign In